LSAT and Law School Admissions Forum

Get expert LSAT preparation and law school admissions advice from PowerScore Test Preparation.

User avatar
 Dave Killoran
PowerScore Staff
  • PowerScore Staff
  • Posts: 5852
  • Joined: Mar 25, 2011
|
#26552
Complete Question Explanation
(The complete setup for this game can be found here: lsat/viewtopic.php?t=11091)

The correct answer choice is (E)

If M is scheduled for the second performance, then from the last rule P must be scheduled for the fifth performance (and L cannot be scheduled for the third performance). This information eliminates answer choices (C) and (D).

Answer choices (A) and (B) can be eliminated because with P performing fifth, the KJ block cannot be placed so J is sixth or K is fourth. Hence, answer choice (E) is correct.
User avatar
 LeslyLSATinLA
  • Posts: 16
  • Joined: Jan 30, 2023
|
#99359
I'm still a little confused on how E is correct. I chose D. Is E correct because P being fifth forces O at 4th and the KJ block at slots 6 and 7?Can someone clarify this?
 Adam Tyson
PowerScore Staff
  • PowerScore Staff
  • Posts: 5153
  • Joined: Apr 14, 2011
|
#99373
As the explanation at the top of the thread explains, if M is 2nd, P must be 5th, since P can only ever be 2nd or 5th according to the last rule of the game. That already eliminates answer D - N cannot be 5th because P is 5th! It has nothing to do with O or the KJ block.

Answer E is correct simply because it is the only answer that could be true. All the others would end up violating some rule of the game!

To illustrate, here are some possible solutions when M is 2nd:

OMKJPLN
LMNOPKJ
OMNLPKJ
NMKJPOL

There are others, but you can see that O is NOT forced to be 4th and the KJ block is NOT forced to be 6th and 7th, even though those are all options. But K cannot be 4th, since that would force J to be 5th, which cannot occur since P is 5th. J cannot be 6th for the same reason - that would make K 5th, but P is 5th. L cannot be 3rd because that would require N to be 5th, but it isn't 5th - P is.

This isn't a Must Be True question, and O doesn't have to be 4th. It's a Could Be True question, and O could be 4th!

Get the most out of your LSAT Prep Plus subscription.

Analyze and track your performance with our Testing and Analytics Package.